Pleasee help mee
a circular coil of 100 turns and cross-sectional area of 2. 0 cm² carrying a 50 mA current is placed in a magnetic field of 0. 5 T parallel to the plane of the coil. Calculate the torque acting on the coil?

Answers

Answer 1

A circular coil of 100 turns and a cross-sectional area of 2. 0 cm² carrying a 50 mA current is placed in a magnetic field of 0. 5 T parallel to the plane of the coil. The torque acting on the coil is 0.01 Nm.

The torque acting on a circular coil placed in a magnetic field can be calculated using the formula: [tex]T = NABsin\theta[/tex] , where N is the number of turns in the coil, A is the area of each turn, B is the magnetic field strength, and θ is the angle between the magnetic field and the plane of the coil.

Substituting the given values, we have

[tex]T = (100)(2.0 \times 10^{-4} m^2)(0.5 T)sin90^{\circ}[/tex]

T = 0.01 Nm.

Therefore, the torque acting on the coil is 0.01 Nm.

In this scenario, a magnetic field is acting parallel to the plane of the coil, which results in the maximum torque being produced, and thus, the value of the angle θ is 90°.

The magnetic field generates a force on each turn of the coil, and this force creates a torque that makes the coil rotate around an axis perpendicular to the magnetic field. The greater the number of turns in the coil, the greater the torque produced.

To know more about torque refer here:

https://brainly.com/question/28220969#

#SPJ11


Related Questions

A uniform, 6 m long and 600-N beam, rests on two supports, as shown. The force exerted onthe b eam by the right support B is closest to:

Answers

The force exerted on the beam by the right support B is closest to: (B).320N is correct option.

If the beam is at rest, the sum of the forces and the sum of the torques acting on it must be equal to zero.

Assuming the beam is supported at its two ends, the sum of the forces acting on the beam will be equal to the weight of the beam, which is given by:

W = m * g

W = (600 N) / (9.81 m/s²) ≈ 61.14 kg

Each support will exert an equal and opposite force on the beam, which we can denote as F. Therefore, the sum of the forces acting on the beam will be:

ΣF = 2F - W = 0

Solving for F, we get:

F = W/2

F ≈ 30.57 kg ≈ 300 N

Therefore, the force exerted on the beam by the right support B is closest to 300 N.

The complete question is,

A uniform 400-N beam 6 m long rests on two supports. Support Ais im from the left end of the beam Support B is at the right end of the beam. What is the value in N. of support force exerted on the beam by the left support A? 400 0 320 240 O 160

To know more about beam

https://brainly.com/question/29825179

#SPJ1

A football player is running down the field with a momentum of 0. 567 kg*m/s. The player encounters a force that causes him to stop in 1. 4 seconds.


What is the final momentum of the player?


kg*m/s



What is the change in momentum of the player?


kg*m/s



What is the impulse?


N*s



What is magnitude of the force that brings the player to a stop in this amount of time?

Answers

The final momentum of the player is 0 kgm/s, the change in momentum is -0.567 kgm/s, the impulse is -0.567 N*s, and the magnitude of the force that brings the player to a stop in 1.4 seconds is 0.405 N.

We can use the equation for impulse, which is given by:

[tex]impulse = force * time[/tex]

We can also use the equation for change in momentum, which is given by:

change in momentum = final momentum - initial momentum

First, let's find the final momentum of the player. We know that the initial momentum is 0.567 kgm/s, and the player comes to a stop, so the final momentum is 0 kgm/s.

Therefore, the change in momentum is:

change in momentum = final momentum - initial momentum = 0 - 0.567 = -0.567 kg*m/s

The negative sign indicates that the momentum has decreased.

Now, let's find the impulse. We know that the time it takes for the player to come to a stop is 1.4 seconds, so we can plug that into the equation for impulse:

impulse = force x time

-0.567 kg*m/s = force x 1.4 s

Solving for force, we get:

force = -0.567 kg*m/s ÷ 1.4 s = -0.405 N

The negative sign indicates that the force is acting in the opposite direction of the player's momentum.

To know more about impulse refer here

https://brainly.com/question/30466819#

#SPJ11

An object of mass 6.10 kg has an acceleration a⃗ =(1.31 m/s2 )x^+(-0.673 m/s2 )y^.

A)Three forces act on this object: F⃗ 1 , F⃗ 2 ,and F⃗ 3 .Given that F⃗ 1= (3.06 N ) x^ and F⃗ 2= (-1.62 N ) x^+ (1.73 N ) y^ , find F⃗ 3 .

Express your answers using three significant figures separated by a comma.

Answers

Expressing the answer in three significant figures separated by a comma, we get: F⃗ _3 = (-7.99, -6.00) N = (-10.00 N, -38.88°) using newton second law.

Newton second law calculation.

To find the third force, we can use Newton's second law, which states that the net force on an object is equal to its mass times its acceleration:

F⃗ _net = m⃗ a⃗

where F⃗ _net is the vector sum of all the forces acting on the object.

We can start by finding the vector sum of F⃗ _1 and F⃗ _2:

F⃗ _1 + F⃗ _2 = (3.06 N)x^ + (-1.62 N)x^ + (1.73 N)y^

= (1.44 N)x^ + (1.73 N)y^

Now, we can find the net force by subtracting the vector sum of F⃗ _1 and F⃗ _2 from the mass times acceleration:

F⃗ _3 = m⃗ a⃗ - (F⃗ _1 + F⃗ _2 )

= (6.10 kg)(1.31 m/s^2 x^ - 0.673 m/s^2 y^) - (1.44 N)x^ - (1.73 N)y^

= (7.99 N)x^ - (6.00 N)y^

Therefore, the third force F⃗ _3 has a magnitude of 10.00 N and is directed at an angle of 38.88 degrees below the positive x-axis:

|F⃗ _3| = √[(7.99 N)^2 + (-6.00 N)^2] = 10.00 N

θ = tan⁻¹(-6.00 N / 7.99 N) = -38.88° (measured below the positive x-axis)

Expressing the answer in three significant figures separated by a comma, we get:

F⃗ _3 = (-7.99, -6.00) N = (-10.00 N, -38.88°)

Learn more about newton second law below.

https://brainly.com/question/28441255

#SPJ1

HYSICS What mass of lead has the volume as 1 600 kg of alcohol? the values of densities given in table2.7)​

Answers

Explanation:

Solution:

Density of alcohol = 600kg/m³

In g/cm³ = 600/1000

= 0.60 g/cm³

A coil with 20 turns of wire is wrapped around a tube with a cross-sectional area of 1. 0 m2. A magnetic field


is applied at a right angle at 0. 50 T. If the coil is pulled out of the magnetic field in 5 seconds, what emf is


induced in the coil?

Answers

The emf induced in the coil is 2.0 volts.

To calculate the emf induced in the coil with 20 turns of wire, wrapped around a tube with a cross-sectional area of 1.0 m², and a magnetic field applied at a right angle at 0.50 T, when it is pulled out of the magnetic field in 5 seconds, we can use Faraday's Law of Electromagnetic Induction.

The formula for Faraday's Law is:

emf = -N * (ΔΦ/Δt)

where

emf is the induced electromotive force,

N is the number of turns in the coil (20),

ΔΦ is the change in magnetic flux, and

Δt is the time it takes to change the flux (5 seconds).



First, we need to calculate the change in magnetic flux (ΔΦ). Since the coil is completely pulled out of the magnetic field, the final magnetic flux will be zero.

The initial magnetic flux (Φ_initial) can be calculated using the formula:

Φ_initial = B * A

where

B is the magnetic field strength (0.50 T) and

A is the cross-sectional area of the tube (1.0 m²).

Φ_initial = 0.50 T * 1.0 m²

              = 0.50 Wb (Weber)

Now, we can calculate the change in magnetic flux (ΔΦ):

ΔΦ = Φ_final - Φ_initial

      = 0 Wb - 0.50 Wb

      = -0.50 Wb

Next, we can plug the values into Faraday's Law formula:

emf = -20 * (-0.50 Wb / 5 s)

       = 20 * (0.10 V)

       = 2.0 V

So, the emf induced in the coil is 2.0 volts.

To know more about emf induced  refer here

https://brainly.com/question/16764848#

#SPJ11

How many criteria determine if a naturally occurring object is "magnetic"?

Answers

There are three main criteria that determine if a naturally occurring object is "magnetic"

1. The object must be ferromagnetic (usually iron, nickel, or cobalt), meaning it contains atoms with unpaired electrons that can align themselves with an external magnetic field.

2. The magnetic moments of these atoms must be strong enough to cause the material to be magnetic.

3. The material must have a net magnetic moment, meaning that the magnetic moments of the individual atoms are aligned in the same direction.

While these three criteria are the primary factors that determine whether a naturally occurring object is magnetic, other factors can also influence its magnetism. For example, the temperature and pressure of the material can affect the strength of its magnetic interactions, and the presence of impurities or defects can alter the way its magnetic moments interact.

To know more about magnetic objects, click here;

https://brainly.com/question/26958664

#SPJ11

Read the following passage and find the two errors. Then, choose the answer that corrects the errors. pH is a measure of the concentration of OH− ions in a solution of an acid or base. The pH scale plots the concentration of solutions in a range from 0–16. (2 points) Group of answer choices pH is a measure of the concentration of OH− ions in a solution of water. The pH scale plots the concentration of solutions in a range from 0–12. pH is a measure of the concentration of H+ ions in a solution of an acid or base. The basic scale plots the concentration of solutions in a range from 0–16. pH is a measure of the concentration of OH− ions in a solution of an acid or base. The acid scale plots the concentration of solutions in a range from 0–16. pH is a measure of the concentration of H+ ions in a solution of an acid or base. The pH plots the concentration of solutions in a range from 0–14.

Answers

The two errors in the passage are:

pH is a measure of the concentration of OH− ions in a solution of an acid or base. The pH scale plots the concentration of solutions in a range from 0–16.

The answer that correct both errors is:

pH is a measure of the concentration of H+ ions in a solution of an acid or base. The pH plots the concentration of solutions in a range from 0–14.

A solution's acidity or basicity is determined using the pH scale. A pH of 0-6.9 is considered acidic, a pH of 7 is neutral, and a pH of 7.1-14 is considered basic. The pH scale is logarithmic, which means that each change in pH reflects a tenfold difference in acidity or basicity.

The concentration of hydrogen ions (H+) in a solution determines the pH of the solution. A base is a chemical that reduces the concentration of H+ ions in a solution, whereas an acid raises the concentration of H+ ions in a solution.

Therefore, The  correct answer choice is:

" pH is a measure of the concentration of H+ ions in a solution of an acid or base. The pH plots the concentration of solutions in a range from 0–14."

For more such questions on pH scale, click on:

https://brainly.com/question/14965267

#SPJ11

Particles q1, 92, and q3 are in a straight line.


Particles q1 = -1. 60 x 10-19 C, 92 = +1. 60 x 10-19 C,


and q3 = -1. 60 x 10-19 C. Particles 91 and q2 are


separated by 0. 001 m. Particles q2 and q3 are


separated by 0. 001 m. What is the net force on 92?


Remember: Negative forces (-F) will point Left


Positive forces (+F) will point Right


-1. 60 x 10-19 C


+1. 60 x 10-19


-1. 60 x 10-19 C


91


+ 92


93


0. 001 m


0. 001 m

Answers

The net force on particle 92 is zero, which means it is in equilibrium and not accelerating.

The electric force between charged particles can be calculated using Coulomb's law, which states that the force is proportional to the product of the charges and inversely proportional to the square of the distance between them.

To find the net force on particle 92, we need to calculate the electric forces acting on it due to the other particles in the line. The force on particle 92 due to particle 91 can be calculated using Coulomb's law:

F_92,1 = kq_1q_2 / r²

            = (9x10⁹ Nm²/C²) * (-1.60x10⁻¹⁹ C) * (1.60x10⁻¹⁹ C) / (0.001 m)²

            = -2.30x10⁻¹⁴ N

The force on particle 92 due to particle 93 can also be calculated using Coulomb's law:

F_92,3 = kq_2q_3 / r²

            = (9x10⁹ Nm²/C²) * (-1.60x10⁻¹⁹ C) * (-1.60x10⁻¹⁹ C) / (0.001 m)²

            = 2.30x10⁻¹⁴ N

Since the forces due to particles 91 and 93 are equal in magnitude but opposite in direction, they cancel out each other. As a result, the net force acting on particle 92 is zero.

To know more about the Particles, here

https://brainly.com/question/29023558

#SPJ4

hydroelectric power plants is a clean energy source. which of the following statements explains a disadvantage of using this source?

Answers

Hydroelectric power plants can cause harm to the environment and local ecosystems, especially when dams are built to create reservoirs.

While hydroelectric power is considered a clean energy source because it doesn't produce harmful emissions or pollutants, it does have negative environmental impacts. The construction of dams and reservoirs can cause significant damage to natural habitats, disrupt water flow, and alter the ecology of local rivers and streams.

This can have a detrimental effect on fish populations, migratory patterns, and even erosion of riverbanks. Additionally, the building of dams and reservoirs can lead to the displacement of communities and the loss of cultural heritage sites. The reliance on hydroelectric power can also be affected by climate change as reduced rainfall can lead to lower water levels in reservoirs, impacting the amount of electricity that can be generated.

To know more about the Hydroelectric, here

https://brainly.com/question/2433706

#SPJ1

Write an equation for the strength of the electrostatic force for two charges that are separated by 10 meters. Use the Gizmo to check your equation

Answers

F = k * ([tex]q_1[/tex][tex]q_2[/tex])/[tex]r^{2}[/tex], where F is the strength of the electrostatic force, k is Coulomb's constant (9X[tex]10^{9}[/tex] N*[tex]m^{2}[/tex]/[tex]C^{2}[/tex]), [tex]q_1[/tex] and [tex]q_2[/tex] are the charges, and r is the distance between the charges in meters.

What is Magnetic Force?

Magnetic force is a fundamental force of nature that is exerted between moving charged particles, such as electrons or between a magnetic field and a moving charge. This force can cause a magnetic material to experience a force of attraction or repulsion depending on its orientation with respect to the magnetic field.

Using the Gizmo to check, we can input two charges and a distance of 10 meters to calculate the electrostatic force between them. The result should match the output of the equation F = k * ([tex]q_1[/tex]*[tex]q_2[/tex])/[tex]r^{2}[/tex].

To learn more about Magnetic Force, visit;

https://brainly.com/question/14411049

#SPJ4

2. 2 "However, we need to examine our environment, and ask problematic questions such as,


"whose law?" Obviously, it's the law of the dominant class in that society (Study Guide,


p. 7)


Which class does the South African law, including the constitution, serve? Explain fully. (3)

Answers

The law in South Africa, including the constitution, serves the interests of the dominant class, which historically has been the white minority. During apartheid, the law was used to enforce segregation and discrimination against the majority black population.

While the constitution and laws have since been revised to promote equality and protect human rights, there are still systemic issues that continue to serve the interests of the wealthy and powerful.

For example, land ownership remains highly concentrated in the hands of a few, and the legal system can be slow and expensive, making it difficult for marginalized communities to access justice. Additionally, the legacy of apartheid-era policies and practices continues to impact access to education, healthcare, and economic opportunities for many black South Africans.

Overall, while progress has been made in addressing inequality and promoting social justice, the law in South Africa still reflects the interests of the dominant class and requires continued efforts to ensure that it serves the needs of all citizens.

Know more about human rights here:

https://brainly.com/question/3444313

#SPJ11

A person climbs to the top of the tallest mountain in South America. Since they are farther away from the center of the Earth, how does their weight compare to what it was at the base of the mountain?

Answers

Answer:

A person's weight will be slightly less at the top of a mountain than at the base. This is because the force of gravity is slightly weaker at higher altitudes. The force of gravity is directly proportional to the mass of the Earth and inversely proportional to the square of the distance between the object and the center of the Earth. Since the person is farther away from the center of the Earth at the top of the mountain, the force of gravity is slightly weaker. This means that the person will weigh slightly less.

The amount of weight loss is very small, and it is not something that most people would notice. However, it is a real effect, and it can be measured. In fact, scientists have used this effect to measure the mass of the Earth.

what is the highest temperature allowed for cold holding fresh salsa?

Answers

The highest temperature allowed for cold holding fresh salsa is 41°F (5°C) or below.

What is the highest temperature?

The U.S. Food and Drug Administration (FDA) Food Code stipulates that potentially hazardous foods, such as fresh salsa, must be stored at or below 41°F (5°C) in order to prevent the growth of harmful microorganisms.

It's important to regularly monitor the salsa's temperature and discard any that has been held over this degree for longer than four hours in order to ensure food safety.

Learn more about  temperature:https://brainly.com/question/29628128

#SPJ1

during the collision between a bug and a truck on the freeway, the truck exerts a much larger force on the bug than the one that the bug exerts on the truck. true or false

Answers

The statement "during the collision between a bug and a truck on the freeway, the truck exerts a much larger force on the bug than the one that the bug exerts on the truck" is actually false.

According to Newton's Third Law of Motion, every action has an equal and opposite reaction. In this case, when the bug and the truck collide, both of them exert forces on each other that are equal in magnitude but opposite in direction. While the force has a greater impact on the bug due to its smaller mass, the forces exerted by both the bug and the truck are equal.

Learn more about collision here:-

https://brainly.com/question/13138178

#SPJ11

calculate the wavelengths of the components of the first line of the lyman series, taking the fine structure of the 2p level into account.

Answers

The wavelengths of the components of the first line of the Lyman series, taking the fine structure of the 2p level into account, are 121.4 nm and 121.2 nm for j = 1/2 and j = 3/2, respectively.

The Lyman series is a series of spectral lines in the hydrogen atom's emission spectrum that corresponds to transitions from higher energy levels to the ground state. The first line of the Lyman series corresponds to a transition from the 2p energy level to the 1s energy level.

To take the fine structure of the 2p level into account, we need to use the formula for the energy of a hydrogen atom with fine structure:

[tex]E= -13.6 eV*\frac{1}{n^{2} } *[1+a^{2} * \frac{(\frac{Z}{n}) ^{4}}{(n-j-\frac{1}{2})^{2} } ][/tex]

where n is the principal quantum number, Z is the atomic number (1 for hydrogen), j is the total angular momentum quantum number of the electron, and α is the fine structure constant.

For the 2p energy level, j can be either 1/2 or 3/2, so we need to calculate the wavelengths for both possibilities. Using the formula for the energy of the 2p level and the energy of the 1s level (-13.6 eV), we can calculate the wavelengths of the first line of the Lyman series with fine structure for each case:

For j = 1/2: λ = 121.6 nm * [1 - (1/9) * α²] = 121.6 nm * 0.9988 = 121.4 nm

For j = 3/2: λ = 121.6 nm * [1 - (4/9) * α²] = 121.6 nm * 0.9972 = 121.2 nm

Taking the fine structure of the 2p level into consideration, the wavelengths of the components of the first line of the Lyman series are 121.4 nm and 121.2 nm for j = 1/2 and j = 3/2, respectively.

To know more about the Lyman series, here

https://brainly.com/question/29597178

#SPJ4

the sun is a star in the milky way galaxy. when viewed from the side, the galaxy looks like a disk that is approximately 100,000 light- years in diameter (a light-year is the distance light travels in one year) and about 1000 light-years thick (figure p25.8). what is the diameter and thickness of the milky way in meters? in kilometers? in miles?

Answers

Answer:

1,000,000,000,000,000,000 km (about 100,000 light years or about 30 kpc)

Explanation:

Armatures are wound to provide high voltage, high current, or some specific combination of voltage and current. which type of winding provides moderate voltage and moderate current

Answers

The type of winding that provides moderate voltage and moderate current is called "lap winding." A lap winding connects each armature conductor to the adjacent conductor in a path that runs parallel to the field poles, resulting in multiple parallel paths and a moderate voltage output.

Lap windings are commonly used in direct current (DC) motors and generators, as they provide a balance between high voltage and high current, making them suitable for a range of applications.

The winding is constructed by arranging the armature conductors in concentric circles around the armature core, and then connecting the conductors end-to-end in a continuous loop.

In a lap winding, the number of parallel paths is equal to the number of field poles. This means that a four-pole motor or generator will have four parallel paths, while a six-pole machine will have six parallel paths. The number of parallel paths determines the output voltage and current of the machine, with more parallel paths producing higher output.

In summary, lap winding is a type of armature winding that provides moderate voltage and moderate current, making it suitable for a range of applications. It is constructed by connecting armature conductors in a continuous loop in multiple parallel paths that produce a balanced output.

To know more about lap winding refer here:

https://brainly.com/question/28227580#

#SPJ11

A storage tank contains petrol to a height of 4. 7m. If the pressure at the base of the tank is "32. 3" kPa, determine the density of the petrol. Take the acceleration due to gravity to be 9. 81ms^-2.

Answers

The density  of the petrol in the storage tank is approximately: 703 kg/m³.

To determine the density of the petrol in a storage tank with a height of 4.7m and a pressure at the base of 32.3 kPa, we can use the following formula:

Pressure = Density × Gravity × Height

Where Pressure is given as 32.3 kPa, Gravity is 9.81 m/s² (as provided), and Height is 4.7m. To find the density, we can rearrange the formula as follows:

Density = Pressure / (Gravity × Height)

Now, we can plug in the values:

Density = 32,300 Pa / (9.81 m/s² × 4.7m)

Density ≈ 703 kg/m³

Therefore, the density of the petrol in the storage tank is approximately 703 kg/m³.

To know more about density, refer here:

https://brainly.com/question/28929608#

#SPJ11

Complete question:

A storage tank contains petrol to a height of 4. 7m. If the pressure at the base of the tank is "32. 3" kPa, determine the density of the petrol. Take the acceleration due to gravity to be 9. 81ms⁻².

Leilani Hendricks
4/4/23
Test Name: T-Science-Gr5-T5-PBT (2022-2023)
Test ID: 2710825
1. Sophia rides her bike to and from school. Sophia's bike has a special tape that reflects energy from
the sun to make it easier for cars to see her. She also uses a bell to let other bikers know if she is going
to move pass them. Which of the following form of energy does Sophia not use when biking?
A. mechanical energy
B. sound energy
C. light energy
D. electrical energy

Answers

D. Electrical energy. Sophia does not use electrical energy when biking. The special tape on her bike reflects light energy from the sun to make it easier for cars to see her.

What is Light Energy?

Light energy is a form of electromagnetic radiation that travels through space as waves, and can be perceived by the human eye as colors of the visible spectrum. Light energy can also exist as particles called photons. Light energy is able to travel through transparent or translucent substances, such as air, water, and glass. Light energy plays a crucial role in many natural processes, such as photosynthesis, vision, and the heating of the Earth's atmosphere. It is also widely used by humans in applications such as lighting, telecommunications, and photography.

She uses a bell, which creates sound energy, to let other bikers know if she is going to move past them. The mechanical energy is used by Sophia to pedal the bike and move it forward.

Learn more about Light Energy from the given link

https://brainly.com/question/21288390

#SPJ9

What is the total amount of power delivered to the heater when the heater is connected for 240-volt operation when each heater is 300 watts?

Answers

The total amount of power delivered to the heater when each heater is 300 watts and the heater is connected for 240-volt operation is 300 watts.

To calculate the total amount of power delivered to the heater when each heater is 300 watts and the heater is connected for 240-volt operation, we can use the formula:

P = V * I

where P is power, V is voltage, and I is current.

For a 240-volt operation, we can calculate the current using Ohm's law:

V = I * R

where R is the resistance of the heater.

R can be calculated using the formula:

[tex]R = V^2 / P[/tex]

where P is the power of the heater (in watts).

Substituting the given values, we get:

R = [tex]240^2 / 300[/tex] = 192 Ω

Now, we can calculate the current:

I = V / R = 240 / 192 = 1.25 A

Finally, we can calculate the total power delivered to the heater:

P = V * I = 240 * 1.25 = 300 watts

To know more about power refer here

https://brainly.com/question/29575208#

#SPJ11

Four identical particles of mass 0. 913 kg each are placed at the vertices of a 4. 30 m x 4. 30 m square and held there by four massless rods, which form the sides of the square. What is the rotational inertia of this rigid body about an axis that:



a. Passes through the midpoints of opposite sides and lies in the plane of the square


b passes through the midpoint of one of the sides and is perpendicular to the plane of the square


c. Lies in the plane of the square and passes through two diagonally opposite particles

Answers

To calculate the rotational inertia of this rigid body, we need to use the formula I = Σmr², where I is the rotational inertia, Σm is the sum of the masses of all the particles, and r is the distance of each particle from the axis of rotation.

For part b, the axis of rotation passes through the midpoint of one of the sides and is perpendicular to the plane of the square. This means that the distance of each particle from the axis of rotation is the same, which is the half of the diagonal of the square, given by d/2 = 2.415 m. Thus, the rotational inertia can be calculated as I = 4m(2.415)² = 44.2 kg·m².

For part c, the axis of rotation lies in the plane of the square and passes through two diagonally opposite particles. This means that we need to find the distance of the other two particles from the axis of rotation, which can be calculated using the Pythagorean theorem. The diagonal of the square is given by d = 4.30 m, so the distance of each particle from the axis of rotation is √((d/2)² + (d/2)²) = 3.04 m. Thus, the rotational inertia can be calculated as I = 2m(3.04)² + 2m(2.415)² = 76.8 kg·m².

In summary, the rotational inertia of this rigid body depends on the distribution of the particles and the axis of rotation. For a uniform distribution of particles, the rotational inertia can be calculated using the formula I = Σmr². For part b, where the axis of rotation passes through the midpoint of one of the sides and is perpendicular to the plane of the square, the rotational inertia is 44.2 kg·m².

For part c, where the axis of rotation lies in the plane of the square and passes through two diagonally opposite particles, the rotational inertia is 76.8 kg·m².

To know more about rotational inertia refer here

https://brainly.com/question/22513079#

#SPJ11

Activity 3: musical instruments of mindanao ((moro/islamic musie))
write the different musical solo instruments and musical ensembles in mindanao instrumental music.
bamboo ensemble
kulintang ensemble
membranophones:
1.
2.
1.
2.
3.
metallophones:
1.
2.
3.
4.
5.
string/chordophones
1.
solo instruments
aerophones
1. ​

Answers

In the Moro/Islamic music of Mindanao, there are several solo instruments and ensembles used for musical performances.

Here are some of them:

Musical Ensembles:

1. Bamboo Ensemble - a group of musicians playing bamboo instruments such as flutes, buzzers, and percussion instruments.

2. Kulintang Ensemble - a group of musicians playing a set of small, horizontally laid gongs of different sizes and pitches, accompanied by drums, cymbals, and other percussion instruments.

Membranophones:

1. Dabakan - a large, single-headed cylindrical drum played with both hands.

2. Gandingan - a single-headed, cylindrical drum played with a single stick.

3. Agung - a large, double-headed gong played with a stick.

Metallophones:

1. Kulintang - a set of small, horizontally laid gongs of different sizes and pitches.

2. Gandingan - a set of four large, vertically hung gongs.

3. Agung - a set of two large, double-headed gongs.

4. Sarunay - a small, vertically hung gong.

5. Babandil - a small, single-headed gong.

String/Chordophones:

1. Kudyapi - a two-stringed lute played with a plectrum.

Solo Instruments:

1. Suling - a bamboo flute played solo or in an ensemble.

2. Kulintang a Tiniok - a small, handheld gong played solo or in an ensemble.

Aerophones:

1. Kutiyapi - a two-stringed lute with a bamboo tube resonator and played solo.

To know more about Mindanao refer here

https://brainly.com/question/31614393#

#SPJ11

A spring gun with a spring constant of 250N/m is compressed 5. Ocm. How fast


will a 0. 025kg dart move when it leaves the gun?



0. 13 m/s


0. 50 m/s


1. 5 m/s


5. 0 m/s

Answers

The dart will move at velocity approximately 5.0 m/s when it leaves the gun.

To calculate the speed of the dart, we can use the conservation of energy principle. When the spring is compressed, it has potential energy, which is converted into the kinetic energy of the dart when it is released. The potential energy of the compressed spring can be calculated using the formula: PE = 0.5 * k * x^2, where PE is the potential energy, k is the spring constant (250 N/m), and x is the compression distance (0.05 m).

PE = 0.5 * 250 * (0.05)^2 = 0.3125 J (joules)

Now, we can use the kinetic energy formula to find the speed of the dart: KE = 0.5 * m * v^2, where KE is the kinetic energy, m is the mass of the dart (0.025 kg), and v is the speed. We can rearrange this formula to solve for v:

v = sqrt((2 * KE) / m)

Plugging in the values, we get:

v = sqrt((2 * 0.3125) / 0.025) ≈ 5.0 m/s

Therefore, the speed of the dart when it leaves the gun is approximately 5.0 m/s.

Know more about potential energy of the compressed spring click here:

https://brainly.com/question/14577144

#SPJ11

A uniform disk of radius 0.455 m0.455 m and unknown mass is constrained to rotate about a perpendicular axis through its center. a ring with the same mass as the disk is attached around the disk's rim. a tangential force of 0.227 n0.227 n applied at the rim causes an angular acceleration of 0.109 rad/s2.0.109 rad/s2. find the mass of the disk.

Answers

The mass of the disk is 1.90 kg.We can start by using the formula for torque, which relates torque to angular acceleration and moment of inertia:

τ = Iα

where τ is the torque, I is the moment of inertia, and α is the angular acceleration.

Since the disk is rotating about a perpendicular axis through its center, its moment of inertia can be calculated as:

I_disk = (1/2)MR^2

where M is the mass of the disk and R is its radius.

Similarly, the moment of inertia of the ring can be calculated as:

I_ring = MR^2

where M is the mass of the ring and R is its radius (which is the same as the radius of the disk).

Since the disk and ring have the same mass, we can add their moments of inertia to get the total moment of inertia:

I_total = I_disk + I_ring = (1/2)MR^2 + MR^2 = (3/2)MR^2

Now we can use the given values of torque and angular acceleration to solve for the mass of the disk:

τ = (1/2)MR^2α

0.227 N-m = (1/2)M(0.455 m)^2(0.109 rad/s^2)

Solving for M, we get:

M = 0.227 N-m / [(1/2)(0.455 m)^2(0.109 rad/s^2)] = 1.90 kg

Therefore, the mass of the disk is 1.90 kg.

To know more about the mass of inertia click here:

https://brainly.com/question/12095689

#SPJ11

3. A slinky has a spring constant of 160 N/m. How much work is done on the slinky


to stretch it 0. 50 meters horizontally across a table?

Answers

The work done on the slinky to stretch it 0.50 meters horizontally across a table is 20 J.

The work done on a spring is given by the equation W = (1/2)[tex]kx^{2}[/tex], where W is the work done, k is the spring constant, and x is the distance stretched. Substituting the given values, we get: W = (1/2)(160 N/m)[tex](0.50m)^{2}[/tex], W = 20 J

Therefore, the work done on the slinky to stretch it 0.50 meters horizontally across a table is 20 J.

The work done is equal to the energy stored in the spring as potential energy due to its deformation.

When the slinky is stretched, the work done on it is stored as potential energy in the spring, which can be converted back to work when the spring is released.

To know more about potential energy, refer here:

https://brainly.com/question/24284560#

#SPJ11

A beam of light travels from air into a transparent material. The angle of incidence is 24 ∘ and the angle of refraction is 17 ∘

Answers

Using this equation, the refractive index of a material with an angle of incidence of 24∘ and an angle of refraction of 17∘ was found to be approximately 1.33, consistent with common transparent materials.

When a beam of light passes from one medium into another, its direction changes due to the change in the speed of light in the different media. This phenomenon is known as refraction. The angle of incidence is the angle between the incident ray and the normal to the surface at the point of incidence, and the angle of refraction is the angle between the refracted ray and the normal.

The relationship between the angles of incidence and refraction is given by Snell's law, which states that the ratio of the sines of the angles of incidence and refraction is equal to the ratio of the speeds of light in the two media. Mathematically,

[tex]$\frac{\sin{\theta_1}}{\sin{\theta_2}}=\frac{v_1}{v_2}$[/tex]

where [tex]"\theta_2"[/tex] is the angle of incidence, [tex]"\theta_2"[/tex] is the angle of refraction, [tex]v_1[/tex] is the speed of light in the incident medium (in this case, air), and [tex]v_2[/tex] is the speed of light in the transparent material.

Assuming that the transparent material has a higher refractive index than air, we know that the angle of refraction will be smaller than the angle of incidence. In this case, we are given that [tex]"\theta_1"[/tex] = 24∘ and [tex]"\theta_2"[/tex] = 17∘. We can use Snell's law to find the refractive index of the transparent material.

First, we need to know the speed of light in the air and the speed of light in the transparent material. The speed of light in air is approximately [tex]$3 \times 10^8 \text{ m/s}$[/tex], and the speed of light in the transparent material depends on its refractive index. Let's denote the refractive index of the material by n. Then, we have:

[tex]$\frac{\sin{24^\circ}}{\sin{17^\circ}}=\frac{3 \times 10^8 \text{ m/s}}{v_2}$[/tex]

Solving for [tex]v_2[/tex], we get:

[tex]$v_2 = (3 \times 10^8 \text{ m/s}) \times \frac{\sin{17^\circ}}{\sin{24^\circ}} \approx 2.26 \times 10^8 \text{ m/s}$[/tex]

Next, we can use the relationship between the speed of light and the refractive index to find n:

[tex]$n = \frac{c}{v_2}$[/tex]

where c is the speed of light in a vacuum

Thus,

[tex]$n = \frac{3 \times 10^8 \text{ m/s}}{2.26 \times 10^8 \text{ m/s}} \approx 1.33$[/tex]

This value of the refractive index is close to that of common transparent materials like water and glass.

In summary, when a beam of light travels from air into a transparent material at an angle of incidence of 24∘ and an angle of refraction of 17∘, the refractive index of the material can be found using Snell's law and the relationship between the speed of light and the refractive index. The calculated value of the refractive index is approximately 1.33, which is consistent with that of common transparent materials.

To learn more about refractive index

https://brainly.com/question/30761100

#SPJ4

lighting flashes and you hear a thunder clap 4 seconds later. the velocity of sound is 340 m/s. how far away did the lighting strike

Answers

Answer:

The lightning struck 1,360 meters away

Explanation:

1. List knowns

Speed of sound = 340 m/s

Time = 4 s

2. Find formula that uses above knowns

Speed = Distance / Time

Distance = Time x Speed

3. Substitute

Distance = 4 s x 340 m/s

Distance = 1360 meters

Based on the data table, find the acceleration and then predict the velocity at the time 3. 5 seconds.


A)


The acceleration is 5 m/s/s and the velocity at a time of 3. 5 seconds will be


35 m/s.


B)


The acceleration is 10 m/s/s and the velocity at a time of 3. 5 seconds will


be 35 m/s.


The acceleration is 10 m/s/s and the velocity at a time of 3. 5 seconds will


be 70 m/s


The acceleration is 5 m/5/5 and the velocity at a time of 3. 5 seconds will be


17. 5 m/s

Answers

The acceleration is 5 m/s/s and the velocity at a time of 3.5 seconds will be 17.5 m/s. Option D is correct.

To find the acceleration, we can use the formula a = (vf - vi) / t, where vf is the final velocity, vi is the initial velocity, and t is the time interval. From the given data table, we can see that the initial velocity is 0 m/s and the final velocity at 4 seconds is 20 m/s. Therefore, the acceleration is (20 m/s - 0 m/s) / 4 s = 5 m/s/s.

To predict the velocity at 3.5 seconds, we can use the formula vf = vi + at, where vi is the initial velocity, a is the acceleration, and t is the time interval. Substituting the given values, we get vf = 0 m/s + 5 m/s/s x 3.5 s = 17.5 m/s. Therefore, the predicted velocity at 3.5 seconds is 17.5 m/s. Option D is correct.

To know more about the Acceleration, here

https://brainly.com/question/10820394

#SPJ4

Answer all of these for me, please!! :)
1) if a circuit has 300 ohms of resistance with a 15 v power supply, what current would it draw in amperes?
2) what is the resistance of a circuit that draws 0.02 a with a 300 v power supply?
3)if the resistance of a circuit with 12 a is doubled without changing the voltage, what will be the new current in a?
4)if the resistance of a circuit with 12 a is halved without changing the voltage, what will be the new current in a?

Answers

The current would be: I = V/R = 15 V / 300 Ω = 0.05 A. The resistance would be: R = V/I = 300 V / 0.02 A = 15,000 Ω. The new current would be 6 A. The new current would be 24 A.

1) Using Ohm's law, we can determine the current drawn by the circuit by dividing the voltage by the resistance. So, the current would be: I = V/R = 15 V / 300 Ω = 0.05 A.

2) Again, using Ohm's law, we can determine the resistance of the circuit by dividing the voltage by the current. So, the resistance would be: R = V/I = 300 V / 0.02 A = 15,000 Ω.

3) According to Ohm's law, if the resistance of a circuit is doubled without changing the voltage, the current will be halved. So, the new current would be 6 A.

4) Similarly, if the resistance of a circuit is halved without changing the voltage, the current will be doubled. So, the new current would be 24 A.

In summary, Ohm's law relates the current, voltage, and resistance in an electric circuit. By knowing any two of these values, we can calculate the third value using the formula I = V/R.

To know more about Ohm's law, refer here:

https://brainly.com/question/1247379#

#SPJ11

A small plate has moved away from a large plate. It has moved150,000 meters in 30 million years. It is moving eastward.


1. What is the rate of motion of the small plate? Express your answer in mm/year.


2. Where would the plate be after 1. 5 million years? Express your answer in m

Answers

The small plate is moving eastward at a rate of 5 millimeters per year. After 1.5 million years, the small plate would be 7,500 meters farther away from the large plate.

To find the rate of motion of the small plate, we can divide the distance it moved by the time it took to move that distance:

Rate of motion = distance/time

In this case, the distance is 150,000 meters and the time is 30 million years, which is equivalent to 30,000,000 years. Converting both values to millimeters and years, respectively, we get:

Rate of motion = (150,000 meters) / (30,000,000 years) * (1000 mm/meter) / (1 year/1)

Simplifying this expression, we get:

Rate of motion = 5 mm/year

To find where the small plate would be after 1.5 million years, we can use the formula:

distance = rate of motion * time

Using the rate of motion we calculated in part 1 (5 mm/year) and the given time of 1.5 million years, we get:

distance = (5 mm/year) * (1.5 million years)

Converting the result back to meters, we get:

distance = 7,500 meters

To know more about plate, here

brainly.com/question/16941906

#SPJ4

Other Questions
Which of the following is the dimension of moment of inertia? (a) ML2 (b) MLT-1 (c) L2T 2 (d) L T-1 Find the surface area of the square pyramid (above) using its net (below). Explain the role of the brackets, and how they effects the sum. Provide the answer for both sums. Sum 1 Sum 2 10 + 7 5 + 3 = 10 + 7 (5 + 3) = Question 8 of 15How are Mexico and the United States similar nations?Check all that apply.A. They both share identical cultures on the interior of each country.B. They both have similar military might and culture.C. They both share cultures, especially along the border.D. They both work together as economic partners.SUBMIT Russ placed $8000 into his credit union account paying 6% compounded semiannually (twice a year). How much will be in Russs account in 4 years Explain what Gandhi means by the "cultural conquest" of India. CH3COOC5H11 Draw this structure it is an ester A person places $13000 in an investment account earning an annual rate of 7. 4%, compounded continuously. Using the formula V = Pe^{rt}V=Pe rt , where V is the value of the account in t years, P is the principal initially invested, e is the base of a natural logarithm, and r is the rate of interest, determine the amount of money, to the nearest cent, in the account after 19 years A freezer chest is in the shape of a rectangular prism. Measured on the inside, the chest is 4 feet wide, 2. 5 feet tall, and 2 feet long. How much space is inside to hold frozen foods? Japan has a form of government.O presidentialparliamentaryO totalitarian Please help with this math problem! How you can solve real-life problems involving mean or expected value Eddie's dog weighs 31. 8 kilograms. How many grams are equivalent to 31. 8 kilograms?A). 0318 gramsB) 318 grams() 3,180 gramsD) 31,800 grams someone help i really need help with this what should susan three main talking points be to convince her mom that she should file her own tax return Katrine manages a sleepy night inn. She knows from experience that last-minute customers will call after 8 p. M. Each evening looking for a room and asking the price. Katrine has empowered her staff to offer discounts when the motel is largely vacant, and to quote the standard price when the motel is close to full. She knows her service is ________, meaning that if no one stays in the room, it generates no revenue that evening. xyz medical facility provides medical services to patients in russellville. the design capacity is 40 patients per hour, and the effective capacity is 35 patients per hour. yesterday the medical facility worked for 7 hours and served 200 patients. what is the effective utilization? a. 60% b. 81.63% c. 3600% d. 19% e. 76.78% Can someone help me asap? Its due today!! What was a goal of the Freedom Rides in southern states in the early 1960s?to highlight how few public transportation options were available outside of citiesB to assist local school boards in their efforts to integrate public schoolsto increase voter turnout in state elections and transport voters to the pollsto call attention to the segregation still widespread on interstate buses True or false: it's not important for people who work for you to know the vision you have for your company.